Sei sulla pagina 1di 16

Por que voce deveria ter resolvido

oproblema 2da OBM2007


Vamos fazer um tratado geral sobre razes primitivas e resduos quadr aticos em geral e depois resolveremos
o problema acima. Aproveitamos para tratar de outros assuntos `as vezes esquecidos como o lema de Thue e
apresentamos um teorema novinho do nosso amigo Hensel!
1. Polinomios mod p
Quando trabalhamos em um corpo (como por exemplo, os reais, os racionais e os n umeros mod p), existe
divisao euclidiana de polinomios e fatora c ao unica. A divisao euclidiana vem diretamente do algoritmo da
chave de divisao e o fato de que e possvel dividir por qualquer a 0 (m od. p); a fatora c ao unica vem do
fato de que se f(x) e irredutvel ent ao f(x) | g(x)h(x) f(x) | g(x) ou f(x) | h(x), que por sua vez vem
da divisao euclidiana.
Alem disso, o teorema do resto para polinomios tambem e valido em Z/pZ.
Com tudo isso, vem `a tona o seguinte
Teorema 1.1. Seja f(x) um polinomio com coecientes inteiros e de grau d e p um primo. Ent ao a
congruencia f(x) 0 (mod. p) tem no maximo d razes mod p, contando multiplicidades.
Demonstra cao
Indu c ao em d. Para d = 0, n ao h a razes e para d = 1, o polinomio e da forma ax +b, a 0 (m od. p), cuja
raiz e x = b a
1
mod p.
Seja f(x) de grau d e r uma raiz de f (caso f n ao tenha razes, o teorema est a demonstrado, pois d 0).
Ent ao, pelo teorema do resto, f(x) (xr)g(x) (m od. p), sendo g de grau d1. Pela hipotese de indu c ao,
g tem no maximo d 1 razes, e o resultado segue.
Note que esse resultado n ao e valido para modulos compostos. Por exemplo, x
2
1 0 (m od. 8) tem
4 solu c oes.
Exerccios
01. Seja f(x) um polinomio de coecientes inteiros com grau d. Prove que a congruencia f(x) 0
(m od. p), p primo, tem d solu c oes n ao congruentes mod p se, e somente se, na divisao euclidiana de x
p
x
por f(x), x
p
x = f(x)q(x) +r(x), o resto r(x) tiver todos os seus coecientes m ultiplos de p.
2. Razes primitivas
Primeiro, vamos denir ordem.
Deni cao 2.1. Sejam a e m inteiros primos entre si. A ordem de a mod m, denotada por ord
m
a, e o
menor inteiro positivo d tal que a
d
1 (mod. m).
O seguinte lema e particularmente util.
Lema 2.1. Se a
t
1 (mod. m) ent ao ord
m
a | t. Em particular, ord
m
a | (m).
Demonstra cao
Basta realizar a divisao euclidiana de t por ord
m
a: t = q ord
m
a + r, 0 r < ord
m
a. Ent ao a
t
=
(a
ord
m
a
)
q
a
r
a
r
(m od. m). Como r < ord
m
a e ord
m
a e mnimo, r so pode ser zero, e portanto
ord
m
a | t.
A segunda arma c ao decorre imediatamente do teorema de Euler-Fermat.
Ja temos que ord
m
a (m). Ser a que e possvel ord
m
a = (m). E se isso acontecer, sera que isso e
util?

E t ao util que n umeros com essa propriedade ganham ate um nome especial.
Deni cao 2.2. Dizemos que g e raiz primitiva de m quando ord
m
g = (m).
Vamos entender o porque desse nome demonstrando o proximo teorema, que e bastante util.
Teorema 2.1. Se g e raiz primitiva de m ent ao 1, g, g
2
, . . . , g
(m)1
mod m sao todos os restos primos com
m. Isto e, se mdc(a, m) = 1 ent ao existe i tal que a g
i
(mod. m).
Demonstra cao
Note que, sendo (m) > i j 0, g
i
g
j
(m od. m) g
ij
1 (m od. m). Pelo lema da ordem,
ord
m
g | i j (m) | i j. Como 0 i j < (m), i j = 0 i = j. Isto quer dizer que n ao
aparecem n umeros repetidos mod m entre os (m) n umeros 1, g, g
2
, . . . , g
(m)1
, de modo que eles so podem
ser todos os (m) n umeros primos com m.
Parece promissor, n ao? Pena que nem todos os n umeros admitem razes primitivas.
Teorema 2.2. Os n umeros que admitem razes primitivas sao 2, 4, p
n
e 2p
n
, sendo p primo mpar.
Vamos provar esse teorema em varias partes.
Parte 1. Se m tem dois fatores primos mpares distintos p e q ent ao m n ao admite raiz primitiva.
Demonstra cao
Se m admite dois fatores primos mpares distintos p e q ent ao podemos escrever m = a b, com mdc(a, b) = 1,
a, b > 1, p | a e q | b. Note que p1 | (a) e q 1 | (b). Em particular, (a) e (b) sao ambos pares, ou seja,
(a)/2 e (b)/2 sao ambos inteiros. Assim, sendo x primo com m, x
(a)
1 (m od. a) = x
(a)(b)/2
1
(m od. a). Analogamente, x
(a)(b)/2
1 (m od. b). Como a e b sao primos entre si, conclumos que
x
(a)(b)/2
1 (m od. ab), de modo que ord
m
x (a)(b)/2 = (ab)/2 = (m)/2 < (m); ou seja, m n ao
admite raiz primitiva.
Parte 2. 2 e 4 admitem raiz primitiva, mas 2
n
, n 3 n ao.
Demonstra cao
Primeiro, 1 e 3 sao razes primitivas de 2 e 4, respectivamente. Seja x mpar. Temos x
2
1 (m od. 8) e,
para n 3, x
2
n2
1 = (x
2
1)(x
2
+1)(x
4
+1) . . . (x
2
n3
+1) tem pelo menos n fatores 2, ja que 2
3
| x
2
1
e 2 | x
2
i
+ 1, i = 1, 2, . . . , n 3. Logo x
2
n2
1 (m od. 2
n
), ou seja, ord
2
n x 2
n2
< (2
n
) para todo x
mpar. Assim, 2
n
n ao admite razes primitivas para n 3.
Parte 3. Se 4 | m e m > 4 ent ao m n ao admite raiz primitiva.
Demonstra cao
Seja m = 2
k
, mpar e k 2. O caso = 1 ja foi estudado na parte 2. Nos demais casos, basta repetir a
demonstra c ao da parte 1 com 4 no lugar de p e q sendo um divisor de .
Parte 4. Todo primo mpar p admite raiz primitiva.
Demonstra cao
Essa e a parte mais difcil do teorema e e aqui que utilizaremos o teorema sobre polinomios mod p.
Considere o seguinte algoritmo para encontrar uma raiz primitiva de qualquer primo p:
Algoritmo
(1) Tome a = 2.
(2) Seja d
a
= ord
p
a. Se d
a
= p 1, a e raiz primitiva de p; caso
contrario, tome o menor n umero b que n ao e congruente a algum
a
i
mod p.
(3) Seja d
b
= ord
p
b. Se d
b
= p1, b e raiz primitiva de p; se n ao, tome
m e n tais que mdc(m, n) = 1, m | d
a
, n | d
b
e mn = mmc(d
a
, d
b
)
(por que eles existem?).
(4) Troque a por c = a
da/m
b
d
b
/n
e volte ao passo 2.
Vamos provar que esse algoritmo funciona e, o mais importante, termina. Supondo que termine, ele
funciona porque os sinais de termino sao quando encontramos uma raiz primitiva.
Agora, provemos que o algoritmo termina, o que e mais interessante. Primeiro note que d
b
n ao divide
d
a
, pois se dividisse teramos b
da
1 (m od. p), o que n ao pode ocorrer pois a equa c ao x
da
1 (m od. p)
admite no maximo d
a
solu c oes, que sao 1, a, a
2
, . . . , a
da1
, e b n ao e congruente a algum a
i
. Isso implica
mmc(d
a
, d
b
) > d
a
. Alem disso, seja k = ord
p
c. Ent ao (a
da/m
b
d
b
/n
)
k
1 (m od. p). Elevando ambos
os membros por m, obtemos a
dak
b
d
b
mk/n
1 (m od. p) b
d
b
mk/n
1 (m od. p). Lembrando que
d
b
= ord
p
b, d
b
| d
b
mk/n n | mk. Sendo mdc(m, n) = 1, temos n | k. Analogamente, m | k e, portanto
mn | k. Observando ainda que c
mn
1 (m od. p), temos ord
p
c = mn = mmc(d
a
, d
b
) > d
a
. Isto quer dizer
quem a cada itera c ao do algoritmo a ordem do proximo valor aumenta. Portanto, em algum momento iguala
o seu maximo, que e p 1 (note que a escolha de b no algoritmo depende de ord
p
a = p 1).
Parte 5. Se g e raiz primitiva de p mas n ao de p
2
, ent ao g +p e raiz primitiva de ambos.
Demonstra cao
Seja d = ord
p
2 g. Ent ao g
d
1 (m od. p
2
) = g
d
1 (m od. p) = p 1 | d. Logo, como d = p(p 1) e
d | p(p 1), d = p 1, isto e, g
p1
1 (m od. p
2
).
Para conseguirmos provar essa parte, basta demonstrarmos que (g + p)
p1
1 (m od. p
2
). E esse e
um trabalho para o binomio de Newton! Temos (g +p)
p1
g
p1
+
_
p1
1
_
g
p2
p (m od. p
2
) (voce consegue
ver por que n ao precisamos escrever os demais termos do binomio de Newton?). Substituindo g
p1
1
(m od. p
2
) e desenvolvendo as contas: (g + p)
p1
1 + (p 1)pg
p2
1 pg
p2
(m od. p
2
), que n ao e 1,
pois g
p2
0 (m od. p). Logo g +p e raiz primitiva de p
2
(e de p tambem!)
Parte 6. Se g e raiz primitiva de p e p
2
ent ao e raiz primitiva de p
n
, e portanto p
n
admite raiz primitiva.
Demonstra cao
Indu c ao sobre n. A base de indu c ao (n = 1 e n = 2) est a na hipotese. Suponha que g seja raiz primitiva
de p
n1
. Seja d = ord
p
n g. Ent ao g
d
1 (m od. p
n
) = g
d
1 (m od. p
n1
) = ord
p
n1 g | d =
p
n2
(p 1) | d. Como d | (p
n
) d | p
n1
(p 1), temos d = p
n2
(p 1) ou d = p
n1
(p 1). Para
provar que n ao ocorre o primeiro caso, vamos usar o lema de Hensel:
Lema 2.2. Seja p um primo mpar, a um inteiro e n um inteiro positivo. Se p

a 1, > 0 e p

n
ent ao p
+
a
n
1. Alem disso, se n e mpar, p

a + 1, > 0 e p

n ent ao p
+
a
n
+ 1.
(Dizemos que p

divide exatamente m e denotamos por p

m o fato de que m contem exatamente


fatores primos p.)
Note que como p | g
p1
1 e p
2
n ao divide g
p1
1 (caso contrario, g n ao seria raiz primitiva de p
2
),
p g
p1
1. Assim, pelo lema de Hensel, a maior potencia de p que divide g
p
n2
(p1)
1 = (g
p1
)
p
n2
1 e
p
1+n2
= p
n1
. Assim, p
n
n ao divide g
p
n2
(p1)
1, o que e equivalente a g
p
n2
(p1)
1 (m od. p
n
). Logo
ord
p
n g = p
n1
(p 1) = (p
n
) e, portanto, g e raiz primitiva de p
n
.
Parte 7. 2p
n
admite raiz primitiva.
Demonstra cao
Seja g uma raiz primitiva de p
n
. Note que (2p
n
) = (p
n
) (verique!) e considere g ou g + p
n
, o que
for mpar. Sendo d a ordem desse n umero, que denotaremos por h, h
d
1 (m od. 2p
n
) = h
d
1
(m od. p
n
) = (p
n
) | d. Assim, d = (2p
n
).
Exerccios
02. Sejam x e y inteiros positivos. Prove que existem inteiros positivos m e n tais que m | x, n | y,
mdc(m, n) = 1 e mn = mmc(x, y).
03. H a outra maneira de provar que p admite raiz primitiva. Como? Siga os itens!
(a) Prove que

d|n
(d) = n. Dica: conte de duas maneiras a quantidade de pares (x, d) em que mdc(x, n) =
d.
(b) Prove que se d | p 1 a congruencia x
d
1 (m od. p) tem exatamente d solu c oes distintas mod p.
(c) Seja d um divisor de p 1 e r(d) a quantidade de n umeros mod p com ordem igual a d. Prove que
r(d) (d).
(d) Prove que, na verdade, r(d) = (d). Em particular, p admite (p 1) razes primitivas (isso pode ser
generalizado: se m admite raiz primitiva, ent ao admite ((m)) razes primitivas).
04. (OBM 1995, Problema 5) Encontre todas as fun c oes f: Z
+
R tais que, para todos x, y inteiros n ao
negativos,
f(x)f(y) = f(xy) e f(x + 1019) = f(x)
05. (IMO Shortlist 2001) Seja p > 3 um primo. Prove que existe a com 1 a < p 1 tal que a
p1
1 e
(a + 1)
p1
1 n ao sao divisveis por p
2
.
06. Encontre todos os n umeros inteiros positivos n tais que x
25
x (m od. n) para todo inteiro x.
07. Sendo k um inteiro positivo dado, encontre todos os inteiros positivos n tais que 7
n
+ 1 e m ultiplo de
5
k
.
3. Resduos quadraticos
Deni cao 3.1. Dizemos que c e resduo quadr atico mod m quando existe x inteiro tal que x
2
c
(mod. m).
Podemos reduzir qualquer congruencia quadr atica a encontrar resduos quadr aticos. De fato, sendo
D = b
2
4ac,
ax
2
+bx +c 0 (m od. m) 4a
2
x
2
+ 4abx + 4ac 0 (m od. 4am)
(2ax +b)
2
D (m od. 4am)
Primeiro, vamos nos preocupar somente com c primo com m.
Lema 3.1. Seja d = mdc(c, m), d = k
2
, livre de quadrados, c = dc

, m = dm

. Ent ao x
2
c (mod. m)
tem solu c ao se, e somente se, x
2
c

(mod. m

) tem solu c ao e mdc(, m

) = 1. Note que mdc(c

, m

) = 1.
Demonstra cao
Primeiro, note que x
2
c (m od. m) x
2
= c + mt = k
2
(c

+ m

t). Assim, lembrando que


e livre de quadrados, k | x. Sendo x = kx

, obtemos (x

)
2
= c

+ m

t, de modo que (x

)
2
c

(m od. m

). Reciprocamente, x
2
c

(m od. m

) (xk)
2
k
2
c

(m od. k
2
m

) (xk)
2
c
(m od. m/) = (xk)
2
c (m od. m).
Alem disso, suponha que mdc(, m

) = u > 1. Ent ao, (x

)
2
= c

+ m

t = u | c

= du | c, o que e
um absurdo pois du | m e isso implicaria mdc(c, m) = du > d. De fato, mdc(c

, m

) = 1.
A partir daqui, vamos supor sempre que mdc(c, m) = 1.
Lema 3.2. c e resduo quadr atico de m = p
1
1
p
2
2
p

k
k
se, e somente se, c e resduo quadr atico de p
i
i
,
i = 1, 2, . . . , k.
Demonstra cao
Um dos lados e trivial; o outro e uma aplica c ao do Teorema Chines dos Restos.
Agora a ideia e encontrar criterios para vericar se um n umero e resduo quadr atico modulo potencia
de primo. H a dois casos a considerar.
3.1. Resduos quadraticos mod 2
n
Nesse caso, podemos descrever todos os resduos quadr aticos de modo simples:
Teorema 3.1. Os resduos quadr aticos mod 2
n
sao os n umeros da forma 8k + 1, k inteiro.
Demonstra cao
Primeiro, provemos que os resduos quadr aticos devem ser da forma 8k + 1: para ver isso, basta notar que
(2t + 1)
2
= 8
t(t+1)
2
+ 1 1 (m od. 8).
Se n = 1 ou n = 2, o unico mpar da forma 8k + 1 que e um resduo e 1, que de fato e o unico resduo
quadr atico. Nos demais casos, uma contagem basta: h a, em princpio, 2
n1
resduos possveis (todos os
mpares mod 2
n
). Mas h a repeti c oes: de fato, x
2
y
2
(m od. 2
n
) 2
n
| (x y)(x +y). Como x e y sao
mpares, a maior potencia de 2 que divide ambos (e, portanto, sua soma (x y) +(x +y) = 2x) e 2. Assim,
sendo n > 2, 2
n2
|
xy
2

x+y
2
2
n2
|
xy
2
ou 2
n2
|
x+y
2
x y (m od. 2
n1
) x y
(m od. 2
n
) ou x y + 2
n1
(m od. 2
n
).
Deste modo, cada resduo quadr atico est a sendo contado exatamente quatro vezes, de modo que o total
de resduos quadr aticos mod 2
n
e 2
n1
/4 = 2
n3
. Porem, todos devem ser da forma 8k + 1 e, por uma
grande coincidencia, h a 2
n3
resduos da forma 8k+1 mod 2
n
. Ent ao todo n umero da forma 8k+1 e resduo
quadr atico mod 2
n
.
3.2. Resduos quadraticos mod p
n
, p > 2
Nesse caso, o problema e mais simples, simplesmente porque potencias de primos mpares admitem razes
primitivas (veremos por que mais para frente).
Teorema 3.2. c e resduo quadr atico modulo p
n
se, e somente se, c
(p
n
)/2
1 (mod. p
n
).
Demonstra cao
Se c e resduo quadr atico mod p
n
ent ao existe um inteiro x tal que x
2
c (m od. p
n
) = (x
2
)
(p
n
)/2

c
(p
n
)/2
(m od. p
n
) x
(p
n
)
c
(p
n
)/2
(m od. p
n
) c
(p
n
)/2
1 (m od. p
n
).
Reciprocamente, seja g uma raiz primitiva de p
n
. Ent ao c g
k
(m od. p
n
) para algum k. O nosso
prop osito e provar que k e par, de modo que x = g
k/2
e uma raiz de x
2
(m od. p
n
). Mas isso e simples:
de fato, c
(p
n
)/2
1 (m od. p
n
) g
k(p
n
)/2
1 (m od. p
n
). Como ord
p
n g = (p
n
), conclumos que
(p
n
) | k(p
n
)/2, o que implica que k e par.
Esse ultimo teorema e conhecido como criterio de Euler.
4. O problema 2 da OBM 2007
Com isso, podemos resolver o problema 2 da OBM 2007.
Problema 2, OBM 2007. Para quantos n umeros inteiros c, 2007 c 2007 , existe um inteiro x tal
que x
2
+c e m ultiplo de 2
2007
?
Resolu cao
Poderamos simplesmente aplicar dois dos lemas anteriores (procure-os!), mas vamos dar uma solu c ao difer-
ente, utilizando uma das melhores amigas da Teoria dos N umeros: a indu c ao.
Vamos nos preocupar primeiro com os resduos quadr aticos primos com 2
n
, que sao os mpares. Como
antes, sendo x = 2t + 1 mpar, x
2
= 8
t(t+1)
2
+ 1 1 (m od. 8).
Provemos, por indu c ao, que os resduos quadr aticos mpares mod 2
n
sao todos os inteiros da forma 8k+1.
A base de indu c ao e para n = 3 e e imediata. Alem disso, suponha que m seja resduo quadr atico mod 2
n
,
ou seja, x
2
m (m od. 2
n
). Se x
2
m (m od. 2
n+1
), acabou. Caso contrario, x
2
m+2
n
(m od. 2
n+1
).
Para consertar isso, tome x

= x + 2
n1
. Temos (x

)
2
x
2
+ 2 2
n1
+ 2
2n2
(m + 2
n
) + 2
n
+ 0 m
(m od. 2
n+1
), pois n 2 2n 2 n. Portanto m e resduo quadr atico mod 2
n+1
tambem.
Agora, vamos considerar os resduos quadr aticos pares. Note que se m = 2
2r
, mpar, 2r n 3,
ent ao x
2
m (m od. 2
n
) x
2
= 2
2r
+ 2
n
u = 2
2r
( + 2
n2r
u) = 2
2r
| x
2
2
r
| x. Sendo
x = 2
r
x
0
, x
2
m (m od. 2
n
) x
2
0
(m od. 2
n2r
). Como n 2r 3, deve ser da forma 8k + 1.
Se n = 2
2r+1
, mpar, 2r + 1 < n, ent ao x
2
m (m od. 2
n
) x
2
= 2
2r+1
+ 2
n
u = 2
2r+1
( +
2
n2r1
u). Como + 2
n2r1
u e mpar, n ao existe x satisfazendo tal condi c ao, ja que x
2
deve ter uma
quantidade par de fatores 2.
Falta somente estudar os casos em que m = 2
2r
= 2
n2
ou m = 2
2r
= 2
n1
, mpar. Esses casos
sao testados manualmente: se m = 2
n2
, basta testar m = 2
n2
e m = 3 2
n2
. Ja sabemos que m = 2
n2
e da forma 2
2r
(8k + 1). Uma rapida analise mostra que 3 2
n2
n ao e possvel (fazendo as mesmas contas
anteriores, encontramos x
2
0
3 (m od. 4), o que e uma contradi c ao). Da mesma forma, se m = 2
n1
, basta
somente testar m = 2
n1
, que tambem e da forma 2
2r
(8k + 1).
S o nos resta mostrar que m = 2
2k
(8k+1) e resduo quadr atico mod 2
n
. Mas isso e simples: basta tomar
x = 2
k
m

, em que (m

)
2
8k + 1 (m od. 2
n
).
O resto e uma contagem simples: basta contar todos os n umeros da forma 2
2k
(8k + 1) no intervalo
[2007, 2007]. Isso e razoavelmente rotineiro, ja que 0 k 6. Contando, voce encontra 670 valores.
4.1. O lema de Hensel aplicado para resduos quadraticos
O fato e que existe uma outra versao do lema de Hensel, utilizada bastante em analise em p-adicos, que
enunciamos aqui.
Lema 4.1. Seja f(x) um polinomio de coecientes inteiros, k > 1 um inteiro e p primo. Suponha que
exista r inteiro tal que f(r) 0 (mod. p
k1
). Ent ao
Se f

(r) 0 (mod. p), ent ao existe um unico inteiro t tal que 0 t < p e f(r +tp
k1
) 0 (mod. p
k
),
sendo t denido por t f

(r) (f(r)/p
k1
) (mod. p).
Se f

(r) 0 (mod. p) e f(r) 0 (mod. p


n
) ent ao f(r +tp
k1
) 0 (mod. p
n
) para todo inteiro t.
Se f

(r) 0 (mod. p) e f(r) 0 (mod. p


n
) ent ao f(r +tp
k1
) 0 (mod. p
n
) para todo inteiro t.
Demonstra cao
Parece difcil demonstrar isso, certo? Na verdade, n ao e t ao complicado quanto parece.

E so saber usar o
sinal do somatorio e utilizar, com parcimonia, o binomio de Newton.
Para facilitar as contas, primeiro note que, do binomio de Newton, (r +tp
k1
)
i
=
_
i
0
_
r
i
+
_
i
1
_
r
i1
tp
k1
+
_
i
2
_
r
i2
(tp
k1
)
2
+ +
_
i
i
_
(tp
k1
)
i
. Mas, sendo k 2, 2(k 1) k, assim reduzindo mod p
k
obtemos
(r + tp
k1
)
i
r
i
+i r
i1
tp
k1
(m od. p
k
).
Agora, podemos terminar as contas mais tranquilamente. Fazer as contas em presta c oes e uma boa
pratica: voce consegue estruturar melhor as contas, separar as diculdades individuais de cada parte, e mais
difcil errar e mais facil achar os erros, se esses acontecerem.

E claro que pode ser mais facil juntar as somas
e manipula-las ao mesmo tempo, mas n ao e o caso aqui.
Enm, seja f(x) =

n
i=0
a
i
x
i
. Note que f

(x) =

n
i=1
a
i
ix
i1
. Temos f(r + tp
k1
) =

n
i=0
a
i
(r +
tp
k1
)
i

n
i=0
a
i
(r
i
+ i r
i1
tp
k1
) =

n
i=0
(a
i
r
i
+ a
i
ir
i1
tp
k1
) = f(r) + tp
k1
f

(r) (m od. p
k
). Ent ao,
lembrando que f(r) 0 (m od. p
k1
), temos f(r + tp
k1
) 0 (m od. p
k
) f(r) + tp
k1
f

(r) 0
(m od. p
k
) t f

(r) f(r)/p
k1
(m od. p). Isso prova os tres itens do lema de Hensel.
O lema de Hensel e bastante poderoso; dada a solu c ao de uma congruencia polinomial mod p, ele permite
resolver a mesma congruencia mod p
n
.
Com isso, e possvel, por exemplo, caracterizar todos os resduos quadr aticos mod p
n
, p primo mpar.
Lema 4.2. Existem (p
n
)/2 = p
n1
(p 1)/2 resduos quadr aticos mod p
n
. Em particular, h a (p 1)/2
resduos quadr aticos mod p.
Demonstra cao
Fica a cargo do leitor. (Dica: fa ca uma contagem semelhante ao dos resduos quadr aticos mod 2
n
)
Lema 4.3. Se c e resduo quadr atico mod p ent ao e resduo quadr atico mod p
n
.
Demonstra cao
Uma indu c ao, aliada ao lema de Hensel, basta: seja f(x) = x
2
c. Se r
2
c (m od. p
n
) f(r) 0
(m od. p
n
), sendo f

(r) 0 (m od. p), pelo lema de Hensel existe um unico t, 0 t < p, tal que f(r+tp
n
) 0
(m od. p
n+1
) (r + tp
n
)
2
c (m od. p
n+1
). A base de indu c ao e a propria hipotese do lema, ent ao a
demonstra c ao est a completa.
Teorema 4.1. Sejam a
1
, a
2
, . . . , a
(p1)/2
os resduos quadr aticos mod p. Ent ao os resduos quadr aticos mod
p
n
sao todos os n umeros da forma kp +a
i
, i = 1, 2, . . . , (p 1)/2.
Demonstra cao
Basta combinar os dois lemas anteriores.
Exerccios
08. (IMO 1996, Problema 4) Os inteiros positivos a e b sao tais que os n umeros 15a + 16b e 16a 15b sao
ambos quadrados perfeitos. Ache o menor valor possvel que o menor destes dois quadrados pode assumir.
09. Encontre todos os inteiros positivos k tais que existe um inteiro a tal que (a + k)
3
a
3
e m ultiplo de
2007.
10. (OBM 1996, Problema 6) Seja T(x) = x
3
+14x
2
2x+1. Denotamos T
n
(x) = T(T
n1
(x)) e T
0
(x) = x.
Prove que existe um inteiro positivo N tal que T
N
(x) x e divisvel por 101 para todo inteiro x. Dica:
Prove que T(x) T(y) (mod. 101) x y (mod. 101) e aplique casa dos pombos.
11. (Generalizando o criterio de Euler) Seja p um primo mpar e c e k > 1 inteiros. Prove que existe um
inteiro x tal que x
k
c (m od. p
n
) se, e somente se, c
(p
n
)/d
1 (m od. p
n
), sendo d = mdc(k, (p
n
)).
12. Encontre todos os n umeros inteiros c tais que existe um inteiro x tal que x
3
c e m ultiplo de 3
2007
.
13. (Vietn a 2000) Seja p(x) = x
3
+ 153x
2
111x + 38. Mostre que p(n) e divisvel por 3
2000
para pelo
menos nove inteiros positivos n menores que 3
2000
. Para quantos valores de n e divisvel?
5. O teorema da reciprocidade quadratica
Esse era um dos teoremas favoritos de Gauss. Ele gostava tanto desse teorema que fez nada menos do que
oito demonstra c oes.
Antes, para facilitar, vamos denir uma nota c ao legendr aria para resduos quadr aticos.
Deni cao 5.1. Seja c inteiro e p primo. Denimos o smbolo de Legendre
_
c
p
_
, lido c Legendre p, por
_
c
p
_
=
_
_
_
0 se p | c
1 se c e resduo quadr atico mod p
1 se c n ao e resduo quadr atico mod p
Assim, por exemplo, temos
_
2
7
_
= 1,
_
20
5
_
= 0 e
_
25
p
_
= 1 para todo primo p.
Note que podemos enunciar o criterio de Euler mais diretamente:
Criterio de Euler. Sendo c inteiro e p primo mpar,
_
c
p
_
c
p1
2
(mod. p)
Com isso, voce deve provar sem diculdades as seguintes propriedades:
Teorema 5.1. Sejam a, b inteiros e p primo mpar. Ent ao

_
ab
p
_
=
_
a
p
_

_
b
p
_
Se a b (mod. p),
_
a
p
_
=
_
b
p
_
Agora vamos enunciar outro criterio para resduos quadr aticos, o lema de Gauss.
Lema 5.1. Seja k a quantidade de inteiros j, 1 j (p 1)/2, tais que cj mod p > p/2 (isto e, o resto
da divisao de cj por p e maior que p/2). Ent ao
_
c
p
_
= (1)
k
Demonstra cao
A principal sacada aqui e escrever c
p1
2
mod p de outro modo, utilizando os resduos maiores que p/2. E
utilizamos aqui uma ideia parecida com uma das demonstra c oes do teorema de Euler-Fermat, o gira-gira.
Considere todos os n umeros da forma cj, 1 j (p 1)/2. Note que
p1
2

j=1
cj c
p1
2
_
p 1
2
_
! (m od. p)
Porem, podemos tambem reduzir os fatores mod p antes de multiplica-los. De fato, seja
c
j
= mn{cj mod p, p cj mod p},
de modo que c
j
= pcj mod p se, e somente se, cj mod p > p/2. N ao e difcil provar que c
i
= c
j
i = j:
de fato, c
i
= c
i
i j (m od. p) e, sendo 1 i, j < p/2, devemos ter i = j.
Ao multiplicar os c
j
s mod p, obtemos de novo todos os resduos entre 1 e (p1)/2 (note que os (p1)/2
n umeros multiplicados sao todos diferentes mod p e com restos menores do que p/2). E esse produto e
exatamente igual ao produto dos cjs, com exce c ao das k mudan cas de sinal quando c
j
= p cj mod p.
Assim,
p1
2

j=1
cj (1)
k
p1
2

j=1
c
j
(1)
k
_
p 1
2
_
! (m od. p)
Das duas equa c oes obtidas,
c
p1
2
_
p 1
2
_
! (1)
k
_
p 1
2
_
! (m od. p) c
p1
2
(1)
k
(m od. p),
e, do criterio de Euler, o resultado segue.
E para que serve essa conta? N ao parece ser mais facil do que o criterio de Euler em si! O fato e que
esse resultado e um dos passos decisivos para obtermos a lei da reciprocidade quadr atica.
Vamos dar mais um passo adiante em dire c ao a ela.
Lema 5.2. Se c e mpar e p e primo mpar, ent ao
_
c
p
_
= (1)
M
em que M =
p1
2

j=1
_
cj
p
_
Demonstra cao
O que e exatamente
cj
p
? Simplesmente e o quociente da divisao de cj por p. Ent ao parece interessante
pensarmos na divisao euclidiana de cj por p. Assim, seja r
j
o resto da divisao de cj por p, de modo que
cj = p
cj
p
+ r
j
. Assim, como e de se esperar, vamos fazer uso de uma ideia muito importante na Teoria
dos N umeros: somar tudo e ver o que acontece!
p1
2

j=1
cj = p
p1
2

j=1
_
cj
p
_
+
p1
2

j=1
r
j
Voce reconhece r
j
? N ao e, na verdade, cj mod p? Assim, lembrando da nota c ao do lema de Gauss,
temos c
j
= mn{r
j
, p r
j
}. Lembrando que os c
j
s nada mais sao do que uma permuta c ao dos n umeros 1,
2, . . . , (p 1)/2, temos
p1
2

j=1
j =
p1
2

j=1
c
j
=

rj<p/2
r
j
+

rj>p/2
(p r
j
) =

rj<p/2
r
j

rj>p/2
r
j
+kp
Subtraindo os dois resultados, obtemos
(c 1)
p1
2

j=1
j = p
p1
2

j=1
_
cj
p
_
+ 2

rj>p/2
r
j
kp
Enm, sendo c e p mpares, vendo mod 2 (anal, so estamos interessados em saber se (1)
M
= (1)
k
)
chegamos ao resultado, ja que p 1 (m od. 2) e c 1 0 (m od. 2):
0
p1
2

j=1
_
cj
p
_
k (m od. 2)
p1
2

j=1
_
cj
p
_
k (m od. 2)
Estamos prontos para ver quando 2 e resduo quadr atico de outro primo.
Teorema 5.2. Sendo p primo mpar,
_
2
p
_
= (1)
p
2
1
8
Demonstra cao

E so fazer c = 2 no teorema acima e utilizar a ultima equa c ao antes de utilizarmos que c e mpar:
(2 1)
p1
2

j=1
j = p
p1
2

j=1
_
2j
p
_
+ 2

rj>p/2
r
j
kp
Sendo

p1
2
j=1
j =
1
2
p1
2
p+1
2
= (p
2
1)/8 e 0 < j < p/2 0 < 2j < p
2j
p
= 0, vendo mod 2
obtemos
p
2
1
8
k (m od. 2)
e o resultado segue.
Mais um fato antes do nosso resultado principal:
Lema 5.3. Sejam p e q primos mpares distintos. Ent ao
p1
2

j=1
_
qj
p
_
+
q1
2

k=1
_
pk
q
_
= 1
Demonstra cao
Sejam S = {(a, b) : 1 a (p 1)/2 e 1 b (q 1)/2}, S
1
= {(a, b) S : aq < bp} e S
2
= {(a, b) S :
aq > bp}. Note que, para todos a, b S, ap = bq; assim, S
1
e S
2
formam uma parti c ao de S.
Contemos as cardinalidades de S, S
1
e S
2
. A de S e facil: |S| =
p1
2

q1
2
. Vamos `a de S
1
: xe b.
Ent ao a < bp/q. Note que bp/q < p/2, de modo que h a, ent ao,
bp
q
valores para a, com b xado. Portanto
|S
1
| =

(q1)/2
j=1

jp
q
. Analogamente, |S
2
| =

(p1)/2
k=1

kq
p
. Substituindo em |S| = |S
1
| + |S
2
|, o resultado
segue.
Voce pode se sentir mais `a vontade com esse fato observando a seguinte gura, que exibe a identidade
acima para p = 7 e q = 11:
E chegamos enm `a celebrada lei da reciprocidade quadr atica:
Teorema 5.3. Sejam p e q primos mpares. Ent ao
_
p
q
_

_
q
p
_
= (1)
p1
2

q1
2
Demonstra cao
Basta juntar os lemas anteriores: sendo M =

p1
2
j=1
_
qj
p
_
e N =

q1
2
k=1
_
pk
q
_
= 1 e lembrando que M +N =
p1
2

q1
2
,
_
p
q
_
= (1)
N
e
_
q
p
_
= (1)
M
=
_
p
q
_

_
q
p
_
= (1)
M+N
= (1)
p1
2

q1
2
Isto quer dizer que o problema de saber se c e resduo quadr atico modulo qualquer n umero est a com-
pletamente resolvido.
Exemplo 5.1.
Vericar se 1028 e resduo quadr atico de 2008.
Resolu cao
Primeiro, note que mdc(1028, 2008) = 4. Ent ao basta vericar se 1028/4 = 257 e resduo quadr atico de
2008/4 = 502. Como 502 = 2 251, basta ver se 257 e resduo quadr atico de 2 e 251. De 2 certamente e.
Vejamos se e 251:
_
257
251
_
=
_
6
251
_
=
_
2
251
__
3
251
_
Poderamos calcular 2
125
e 3
125
modulo 251, mas para que correr o risco de errar conta? Basta usar a
reciprocidade quadr atica: para o 2,
_
2
251
_
= (1)
251
2
1
8
= (1)
250252
8
= (1)
125126
= 1
e para o 3
_
3
251
_

_
251
3
_
= (1)
31
2

2511
2

_
3
251
_

_
2
3
_
= 1
_
3
251
_
= 1
Assim,
_
257
251
_
= 1 (1) = 1 e, portanto, 1028 n ao e resduo quadr atico de 2008.
Exerccios
14. Prove que 2
509
1 (m od. 1019) sem calcular 2
509
mod 1019.
15. Prove que se 2
2
n
+ 1 e primo, ent ao todo n umero inteiro e seu resduo quadr atico ou raiz primitiva.
16. Prove que F
n
= 2
2
n
+ 1 e primo se, e somente se, 3
Fn1
2
1 (m od. F
n
).
17. Encontre o conjunto de todos os divisores primos de f(x) = x
2
+ 2, sendo x inteiro.
18. Encontre o conjunto de todos os divisores primos de f(x) = x
2
15, sendo x inteiro.
19. Existe um quadrado perfeito com seus quatro ultimos algarismos n ao nulos e iguais?
20. Seja f(x) = (x
2
13)(x
2
17)(x
2
221). Prove que f(x) 0 (m od. n) sempre tem solu c ao.
21. Seja p = 4k + 1 um primo, k inteiro positivo. Prove que k
k
1 e m ultiplo de p.
6. Outras potencias mod p
Agora, vamos estudar algumas somas.
Teorema 6.1. Seja S
k
= 1
k
+ 2
k
+ 3
k
+ + (p 1)
k
. Ent ao
S
k

_
p 1 (mod. p) se p 1 | k
0 (mod. p) caso contrario
Demonstra cao
Se p 1 | k ent ao i
k
1 (m od. p) para i = 1, 2, . . . , p 1 e, portanto, S
k
p 1 (m od. p).
Caso contrario, seja g uma raiz primitiva de p. Ent ao note que os n umeros 1, g, g
2
, . . . , g
p2
sao todos
os n umeros 1, 2, . . . , p 1 mod p. Assim, como g
k
1 (m od. p),
S
k
1
k
+g
k
+g
2k
+ +g
(p2)k
=
g
(p1)k
1
g
k
1
0 (m od. p)
E se quisermos saber os resduos modulo p
2
ou uma potencia maior? Nesse caso, utilizamos binomio de
Newton ou a sua generaliza c ao:
Teorema 6.2. Seja real. Sendo
_

k
_
=
_
1 se k = 0
(1)(2)(k+1)
k!
se k > 0
ent ao
(x +y)

k=0
_

k
_
x
k
y
k
Demonstra cao

E so expandir (1 +y/x)

na serie de Taylor.
Exemplo 6.1.
Prove que o numerador de S
1
, quando este e escrito em forma de fra c ao irredutvel, e m ultiplo de p
2
, para
p > 3.
Resolu cao
Temos S
1
=
1
1
+
1
2
+
1
3
+ +
1
p1
. Como usar binomio de Newton nesse caso? Basta usar em (p i)
1
:
(p i)
1
=

k=0
_
1
i
_
p
k
(i)
1k
(i)
1
+ (1)p(i)
2
(m od. p
2
)
Parece uma grande roubalheira, mas est a certo! De fato, sendo i
1
o inverso de i mod p
2
,
(p i)(i
1
pi
2
) pi
1
p
2
i
2
+ii
1
+pii
2
pi
1
+ 1 +pi
1
1 (m od. p
2
)
Assim, somando todas as congruencias
(p i)
1
+i
1
pi
2
(m od. p
2
)
obtemos
2S
1
p
p1

i=1
i
2
(m od. p
2
)
Basta ent ao provar que
p
p1

i=1
i
2
0 (m od. p
2
)
p1

i=1
i
2
0 (m od. p),
o que e verdade, ja que p 1 n ao divide 2 (isso, e claro, se p > 3).
Um pequeno detalhe e que zemos a transi c ao de i
1
ser o inverso de i mod p
2
para ser o inverso de
i mod p. Isso e verdade, mas o caminho contrario n ao pode ser tomado. Ent ao tome cuidado ao trabalhar
com inversos em diversos modulos!
Provemos agora o teorema de Wolstenholme:
Teorema 6.3. Seja p > 3 primo. Ent ao
_
2p 1
p 1
_
1 (mod. p
3
)
Demonstra cao

E so abrir:
_
2p 1
p 1
_
=
2p 1
p 1

2p 2
p 2

2p 3
p 3
. . .
p + 1
1
=
_
1 +
p
p 1
__
1 +
p
p 2
__
1 +
p
p 3
_

_
1 +
p
1
_
Lembrando que (x + x
1
)(x + x
2
) (x + x
n
) = x
n
+
1
x
n1
+
2
x
n2
+ +
n
, sendo
k
a soma de
todos os
_
n
k
_
produtos de k n umeros distintos entre x
1
, x
2
, . . . , x
n
,
_
2p 1
p 1
_
= 1
p1
+
_
p
p 1
+
p
p 2
+
p
p 3
+ +
p
1
_
1
p2
+
2
1
p3
+ +
p1
Utilizando a famosa identidade
2
= ((
1
)
2
s
2
)/2 e que os denominadores de
k
sao m ultiplos de p
3
para k 3, temos
_
2p 1
p 1
_
= 1 +pS
1
+
p
2
2
(S
2
1
2S
2
) +
p
3
A
B
em que
S
1
=
1
p 1
+
1
p 2
+
1
p 3
+ +
1
1
e S
2
=
1
(p 1)
2
+
1
(p 2)
2
+
1
(p 3)
2
+ +
1
1
2
Como ja provamos que S
1
tem numerador m ultiplo de p
2
e S
2
tem numerador m ultiplo de p, o resultado
segue.
Exerccios
22. Seja p > 3 um primo. Sendo
1
1
3
+
1
2
3
+
1
3
3
+ +
1
(p 1)
3
=
m
n
com m e n inteiros primos entre si, prove que p
3
divide m.
23. Seja p > 3 um primo. Sendo
1
1
p
+
1
2
p
+
1
3
p
+ +
1
(p 1)
p
=
m
n
com m e n inteiros primos entre si, prove que p
3
divide m.
24. Seja p > 3 um primo. Prove que
_
ap
bp
_

_
a
b
_
(m od. p
3
).
7. O lema de Thue
Muitos dos fatos mais interessantes da Teoria dos N umeros podem ser obtidos utilizando o princpio da casa
dos pombos.
Lema 7.1. Sejam m e n inteiros positivos primos entre si e sejam a e b inteiros positivos tais que ab > n.
Ent ao existe x {1, 2, . . . , a 1} e y {1, 2, . . . , b 1} tais que
mx y (mod. n)
Demonstra cao
Considere as ab expressoes da forma mx +y
m 1 + 1 m 1 + 2 m 1 + 3 m 1 +b
m 2 + 1 m 2 + 2 m 2 + 3 m 2 +b
m 3 + 1 m 3 + 2 m 3 + 3 m 3 +b
.
.
.
.
.
.
.
.
.
.
.
.
.
.
.
m a + 1 m a + 2 m a + 3 m a +b
Como ab > n, existem duas expressoes que deixam o mesmo resto na divisao por n, digamos mx
1
+y
1
e mx
2
+y
2
, com x
1
> x
2
. Assim,
mx
1
+y
1
mx
2
+y
2
(m od. n) m(x
1
x
2
) y
2
y
1
(m od. n)
Note que y
1
= y
2
pois sen ao x
1
= x
2
. Assim, sendo x = x
1
x
2
e y = |y
1
y
2
|, temos 0 < x a 1 e
0 < y b 1 e
mx y (m od. n)
Lema 7.2. Se p = 4k + 1 e primo, existe x tal que x
2
1 (mod. p).
Demonstra cao
Poderamos aplicar diretamente o criterio de Euler (e o resultado seria imediato) para vericar se 1 e resduo
quadr atico ou mesmo tomar diretamente x = ((p 1)/2)! e utilizar o teorema de Wilson ((p 1)! 1
(m od. p)), mas vamos mostrar mais uma aplica c ao interessante da Combinatoria, agora com contagem.
Vamos repartir o conjunto {1, 2, . . . , p 1} em conjuntos da forma C
a
= {a, p a, a
1
, p a
1
}, em que
a
1
e o inverso de a mod p. Note que C
1
= {1, p 1}, pois o inverso de 1 mod p e 1. Alem desse conjunto,
como p 1 = 4k e m ultiplo de 4, deve haver mais um conjunto C
m
com 2 elementos. Isso ocorre quando
m p m
1
(m od. p) m
2
1 (m od. p). Note que n ao pode ocorrer m p m (m od. p) nem
m m
1
(m od. p).
Agora, vamos provar um dos teoremas mais belos da Teoria dos N umeros.
Teorema 7.1. Todo primo da forma p = 4k + 1 pode ser escrito como soma de dois quadrados.
Demonstra cao
Seja m tal que m
2
1 (m od. p) (ele existe pelo lema anterior). Assim, sendo

p
2
> p, pelo lema de
Thue, existem inteiros x {1, 2, . . . ,

p 1} e y {1, 2, . . . ,

p 1} tais que
mx y (m od. p) = m
2
x
2
y
2
(m od. p) x
2
+y
2
0 (m od. p)
Assim, x
2
+y
2
e m ultiplo de p e, como 0 < x, y <

p, 0 < x
2
< p e 0 < y
2
< p, 0 < x
2
+y
2
< 2p. Mas
o unico m ultiplo de p entre 0 e 2p e p, ou seja, p = x
2
+y
2
pode ser escrito como soma de dois quadrados.
Exerccios
Alguns dos exerccios n ao sao exatamente relacionados com os t opicos tratados aqui, mas sao legais, ent ao
decidi coloc a-los assim mesmo!
25. (Romenia 1997) Seja A o conjunto dos inteiros da forma a
2
+2b
2
, sendo a e b inteiros com b = 0. Prove
que se p e primo e p
2
A ent ao p A.
26. Encontre todos os primos que podem ser escritos na forma x
2
+ 3y
2
.
27. Encontre todos os primos que podem ser escritos na forma a
2
+ab +b
2
.
28. Encontre todos os primos que podem ser escritos na forma a
2
+ab b
2
.
29. (OBM 2007, Nvel 2, Problema 3) Mostre que existe um inteiro positivo a tal que
a
29
1
a1
tem pelo menos
2007 fatores primos distintos.
Dica: use mdc, mas use mesmo! Um possvel valor de a e 2
2
2007
.
30. (OBM 2005, Problema 6) Dados a, c inteiros positivos e b inteiro, prove que existe x inteiro positivo
tal que
a
x
+x b (m od. c),
ou seja, existe x inteiro positivo tal que c e um divisor de a
x
+x b.
Dica: use Euler-Fermat e o teorema chines dos restos:
Teorema 7.2. Sejam m
1
, m
2
, . . . , m
k
inteiros primos dois a dois. Ent ao o sistema de congruencias a seguir
admite solu c ao.

x a
1
(mod. m
1
)
x a
2
(mod. m
2
)
.
.
.
x a
k
(mod. m
k
)
31. (Um resultado classico, mas interessante) Sejam f(x) um polinomio n ao constante com coecientes
inteiros e k um inteiro positivo. Prove que existe um inteiro n tal que f(n) = 0 tem pelo menos k fatores
primos distintos.
Dica: Primeiro prove que uma innidade de divisores primos divide algum f(n), da seguinte maneira:
suponha que o conjunto dos primos que dividam todos os f(n)s seja nito; considere um t inteiro tal f(t) = 0
e seja N tal que os expoentes dos primos p que dividem f(t) sejam maiores em N! do que em f(t); ent ao, os
expoentes de todos os primos p
i
que dividem f(N! +t) = f(t) +M N! sao limitados pelo expoente de p
i
em
f(t); nalize com casa dos pombos para provar que f(x) = k tem innitas solu c oes para algum k, chegando
a uma contradi c ao; para fechar, e so usar o teorema chines dos restos.
8. Referencias bibliogracas
[1] Como ja virou costume, muitos dos problemas foram extrados do Mathlinks e do site do John Scholes. . .
http://www.mathlinks.ro/
http://www.kalva.demon.co.uk/
[2] . . . exceto, e claro, os problemas mais recentes (apos 1996) da OBM, que sao do site ocial da OBM:
http://www.obm.org.br/
[3] Um grande livro de Teoria dos N umeros: Fundamental Number Theory with Applications, de Richard
Mollin. Foi a base da parte sobre resduos quadr aticos e teorema de Thue.
[4] Mais um livro legal sobre primos: N umeros primos: misterios e recordes, do grande especialista de
Teoria dos N umeros (e brasileiro!) Paulo Ribenboim. A demonstra c ao de que existe raiz primitiva
modulo primo mpar e de la.
[5] A Wikipedia tem alguns artigos legais sobre Teoria dos N umeros! O nosso novo lema de Hensel foi
extrado de
http://en.wikipedia.org/wiki/Hensels lemma/
[6] Voce pode encontrar mais sobre razes primitivas (e um monte de outros fatos da Teoria dos N umeros)
na Eureka! 2: Divisibilidade, Congruencias e Aritmetica M odulo n, de Carlos G. T. de A. Moreira.
[7] Voce pode encontrar mais sobre reciprocidade quadr atica na Eureka! 15: Reciprocidade Quadr atica,
Carlos Gustavo T. de A. Moreira e Nicolau Cor c ao Saldanha.

Potrebbero piacerti anche